Difference between revisions of "2003 AIME II Problems/Problem 8"

(Added an alternate Solution)
(Continued to add solution)
(11 intermediate revisions by 9 users not shown)
Line 1: Line 1:
== Problem ==
+
==Problem==
 
Find the eighth term of the sequence <math>1440,</math> <math>1716,</math> <math>1848,\ldots,</math> whose terms are formed by multiplying the corresponding terms of two arithmetic sequences.
 
Find the eighth term of the sequence <math>1440,</math> <math>1716,</math> <math>1848,\ldots,</math> whose terms are formed by multiplying the corresponding terms of two arithmetic sequences.
  
== Solution ==
+
==Solution 1==
 
If you multiply the corresponding terms of two arithmetic sequences, you get the terms of a quadratic function. Thus, we have a quadratic <math>ax^2+bx+c</math> such that <math>f(1)=1440</math>, <math>f(2)=1716</math>, and <math>f(3)=1848</math>. Plugging in the values for x gives us a system of three equations:
 
If you multiply the corresponding terms of two arithmetic sequences, you get the terms of a quadratic function. Thus, we have a quadratic <math>ax^2+bx+c</math> such that <math>f(1)=1440</math>, <math>f(2)=1716</math>, and <math>f(3)=1848</math>. Plugging in the values for x gives us a system of three equations:
  
Line 11: Line 11:
 
<math>9a+3b+c=1848</math>
 
<math>9a+3b+c=1848</math>
  
Solving gives a=-72, b=492, and c=1020. Thus, the answer is <math>-72(8)^2+492\cdot8+1020=348</math>
+
Solving gives <math>a=-72, b=492,</math> and <math>c=1020</math>. Thus, the answer is <math>-72(8)^2+492\cdot8+1020= \boxed{348}.</math>
  
==Alternate Solution==
+
== Solution 2 ==
 
Setting one of the sequences as <math>a+nr_1</math> and the other as <math>b+nr_2</math>, we can set up the following equalities
 
Setting one of the sequences as <math>a+nr_1</math> and the other as <math>b+nr_2</math>, we can set up the following equalities
  
Line 38: Line 38:
 
Now we find
 
Now we find
  
<math>(a+7r_1)(b+7r_2) = ab + 7(ar_2 + br_1) + 49r_1r_2 = 1440 + 7(348) + 49(-72) = \boxed{348}</math>
+
<math>(a+7r_1)(b+7r_2) = ab + 7(ar_2 + br_1) + 49r_1r_2 = 1440 + 7(348) + 49(-72) = \boxed{348}</math>.
  
== See also ==
+
==Solution 3==
 +
Let the first sequence be
 +
 
 +
<math>A={a+d_1, a + 2d_1, a + 3d_1, \cdots}</math>
 +
 
 +
and the second be
 +
 
 +
<math>B={b+d_2, b + 2d_2, b + 3d_2, \cdots}</math>,
 +
 
 +
with <math>(a+d_1)(b+d_2)=1440</math>. Now, note that the <math>n^{\text{th}}</math> term of sequence <math>A</math> is <math>a+d_1 n</math> and the <math>n^{\text{th}}</math> term of <math>B</math> is <math>b + d_2
 +
n</math>. Thus, the <math>n^{\text{th}}</math> term of the given sequence is
 +
 
 +
<math>n^2(d_1 + d_2) + n(ad_2 + bd_1) + ab</math>,
 +
 
 +
a quadratic in <math>n</math>. Now, letting the given sequence be <math>C</math>, we see that
 +
 
 +
<math>C_n - C_{n-1} = n^2(d_1 + d_2) + n(ad_2 + bd_1) + ab - (n-1)^2(d_1 + d_2) - (n-1)(ad_2 + bd_1) - ab = n(2d_1 + 2d_2) + ad_2 + bd_1 - d_1 - d_2</math>,
 +
 
 +
a linear equation in <math>n</math>! Since <math>C_2 - C_1 = 276</math> and <math>C_3 - C_2 = 132</math>, we can see that, in general, we have
 +
 
 +
<math>C_n - C_{n-1} = 420 - 144n</math>.
 +
 
 +
Thus, we can easily find
 +
 
 +
<math>C_4 - C_3 = -12 \rightarrow C_4 = 1836</math>,
 +
 
 +
<math>C_5 - C_4 = -156 \rightarrow C_5 = 1680</math>,
 +
 
 +
<math>C_6 - C_5 = -300 \rightarrow C_6 = 1380</math>,
 +
 
 +
<math>C_7 - C_6 = -444 \rightarrow C_7 = 936</math>, and finally
 +
 
 +
<math>C_8 - C_7 = -588 \rightarrow \boxed{C_8 = 348}</math>.
 +
 
 +
==Solution 4(Tedious) ==
 +
Start by labeling the two sequences:
 +
 
 +
Sequence 1:<math>a,a+d_1,a+2d_1,\dots a+(n-1)d_1</math>,
 +
 
 +
Sequence 2:<math>b,b+d_2,b+2d_2,\dots b+(n-1)d_2</math>.
 +
 
 +
Additionally, label the sequence given in the problem the function <math>f</math>, such that
 +
 
 +
<math>f(1)=1440,f(2)=1716,f(3)=1848</math>.
 +
 
 +
Then,
 +
 
 +
<math>f(1)=ab,</math>
 +
 
 +
<math>f(2)=(a+d_1)(b+d_2)=a+ad_2+d_1b+d_1d_2,</math>
 +
 
 +
<math>f(3)=(a+2d_1)(b+2d_2)=ab+2ad_2+2bd_1+4d_1d_2,</math>
 +
 
 +
and <math>f(8)=(a+7d_1)(b+7d_2)=ab+7ad_2+7bd_1+49d_1d_2</math>.
 +
 
 +
In order to find <math>f(8)</math> add <math>f(3)</math> enough times to get the difference between the <math>d_1d_2</math> and <math>ad_2+bd_1</math> terms, then add <math>f(2)</math> and <math>f(1)</math> to get the other terms:
 +
 
 +
<math>21f(3)=21ab+42ad_2+42bd_1+84d_1d_2</math>
 +
 
 +
<math>21f(3)-35f(2)=21ab+42ad_2+42bd_1+84d_1d_2-35a-35ad_2-35d_1b-35d_1d_2=-14ab+7ad_2+7bd_1+49d_1d_2</math>
 +
 
 +
<math>21f(3)-35f(2)+15f(1)=-14ab+7ad_2+7bd_1+49d_1d_2+15ab=ab+7ad_2+7bd_1+49d_1d_2</math>
 +
 
 +
Now that the expression is in terms of the given values, insert values and solve:
 +
 
 +
<math>21*1848-35*1716+15*1440=1848+20*132+(20*1716-35*1716+15*1716)+15*(-276)</math>
 +
 
 +
<math>=1848+5*132+15(132-276)</math>
 +
 
 +
<math>=1848+660+15(-144)</math>
 +
 
 +
<math>=348</math>
 +
 
 +
==See also ==
 
{{AIME box|year=2003|n=II|num-b=7|num-a=9}}
 
{{AIME box|year=2003|n=II|num-b=7|num-a=9}}
 +
 +
[[Category: Intermediate Algebra Problems]]
 +
{{MAA Notice}}

Revision as of 15:31, 24 June 2021

Problem

Find the eighth term of the sequence $1440,$ $1716,$ $1848,\ldots,$ whose terms are formed by multiplying the corresponding terms of two arithmetic sequences.

Solution 1

If you multiply the corresponding terms of two arithmetic sequences, you get the terms of a quadratic function. Thus, we have a quadratic $ax^2+bx+c$ such that $f(1)=1440$, $f(2)=1716$, and $f(3)=1848$. Plugging in the values for x gives us a system of three equations:

$a+b+c=1440$

$4a+2b+c=1716$

$9a+3b+c=1848$

Solving gives $a=-72, b=492,$ and $c=1020$. Thus, the answer is $-72(8)^2+492\cdot8+1020= \boxed{348}.$

Solution 2

Setting one of the sequences as $a+nr_1$ and the other as $b+nr_2$, we can set up the following equalities

$ab = 1440$

$(a+r_1)(b+r_2)=1716$

$(a+2r_1)(b+2r_2)=1848$

We want to find $(a+7r_1)(b+7r_2)$

Foiling out the two above, we have

$ab + ar_2 + br_1 + r_1r_2 = 1716$ and $ab + 2ar_2 + 2br_1 + 4r_1r_2 = 1848$

Plugging in $ab=1440$ and bringing the constant over yields

$ar_2 + br_1 + r_1r_2 = 276$

$ar_2 + br_1 + 2r_1r_2 = 204$

Subtracting the two yields $r_1r_2 = -72$ and plugging that back in yields $ar_2 + br_1 = 348$

Now we find

$(a+7r_1)(b+7r_2) = ab + 7(ar_2 + br_1) + 49r_1r_2 = 1440 + 7(348) + 49(-72) = \boxed{348}$.

Solution 3

Let the first sequence be

$A={a+d_1, a + 2d_1, a + 3d_1, \cdots}$

and the second be

$B={b+d_2, b + 2d_2, b + 3d_2, \cdots}$,

with $(a+d_1)(b+d_2)=1440$. Now, note that the $n^{\text{th}}$ term of sequence $A$ is $a+d_1 n$ and the $n^{\text{th}}$ term of $B$ is $b + d_2  n$. Thus, the $n^{\text{th}}$ term of the given sequence is

$n^2(d_1 + d_2) + n(ad_2 + bd_1) + ab$,

a quadratic in $n$. Now, letting the given sequence be $C$, we see that

$C_n - C_{n-1} = n^2(d_1 + d_2) + n(ad_2 + bd_1) + ab - (n-1)^2(d_1 + d_2) - (n-1)(ad_2 + bd_1) - ab = n(2d_1 + 2d_2) + ad_2 + bd_1 - d_1 - d_2$,

a linear equation in $n$! Since $C_2 - C_1 = 276$ and $C_3 - C_2 = 132$, we can see that, in general, we have

$C_n - C_{n-1} = 420 - 144n$.

Thus, we can easily find

$C_4 - C_3 = -12 \rightarrow C_4 = 1836$,

$C_5 - C_4 = -156 \rightarrow C_5 = 1680$,

$C_6 - C_5 = -300 \rightarrow C_6 = 1380$,

$C_7 - C_6 = -444 \rightarrow C_7 = 936$, and finally

$C_8 - C_7 = -588 \rightarrow \boxed{C_8 = 348}$.

Solution 4(Tedious)

Start by labeling the two sequences:

Sequence 1:$a,a+d_1,a+2d_1,\dots a+(n-1)d_1$,

Sequence 2:$b,b+d_2,b+2d_2,\dots b+(n-1)d_2$.

Additionally, label the sequence given in the problem the function $f$, such that

$f(1)=1440,f(2)=1716,f(3)=1848$.

Then,

$f(1)=ab,$

$f(2)=(a+d_1)(b+d_2)=a+ad_2+d_1b+d_1d_2,$

$f(3)=(a+2d_1)(b+2d_2)=ab+2ad_2+2bd_1+4d_1d_2,$

and $f(8)=(a+7d_1)(b+7d_2)=ab+7ad_2+7bd_1+49d_1d_2$.

In order to find $f(8)$ add $f(3)$ enough times to get the difference between the $d_1d_2$ and $ad_2+bd_1$ terms, then add $f(2)$ and $f(1)$ to get the other terms:

$21f(3)=21ab+42ad_2+42bd_1+84d_1d_2$

$21f(3)-35f(2)=21ab+42ad_2+42bd_1+84d_1d_2-35a-35ad_2-35d_1b-35d_1d_2=-14ab+7ad_2+7bd_1+49d_1d_2$

$21f(3)-35f(2)+15f(1)=-14ab+7ad_2+7bd_1+49d_1d_2+15ab=ab+7ad_2+7bd_1+49d_1d_2$

Now that the expression is in terms of the given values, insert values and solve:

$21*1848-35*1716+15*1440=1848+20*132+(20*1716-35*1716+15*1716)+15*(-276)$

$=1848+5*132+15(132-276)$

$=1848+660+15(-144)$

$=348$

See also

2003 AIME II (ProblemsAnswer KeyResources)
Preceded by
Problem 7
Followed by
Problem 9
1 2 3 4 5 6 7 8 9 10 11 12 13 14 15
All AIME Problems and Solutions

The problems on this page are copyrighted by the Mathematical Association of America's American Mathematics Competitions. AMC logo.png